Difference between revisions of "1976 AHSME Problems/Problem 25"

(Created page with "== Problem == For a sequence <math>u_1,u_2\dots</math>, define <math>\Delta^1(u_n)=u_{n+1}-u_n</math> and, for all integer <math>k>1, \Delta^k(u_n)=\Delta^1(\Delta^{k-1}(u_n...")
 
Line 10: Line 10:
 
\textbf{(D) }\text{if }k=4,\text{ but not if }k=3\qquad\\
 
\textbf{(D) }\text{if }k=4,\text{ but not if }k=3\qquad\\
 
\textbf{(E) }\text{for no value of }k</math>
 
\textbf{(E) }\text{for no value of }k</math>
 +
 +
== Solution ==
 +
Note that
 +
<cmath>\begin{align*}
 +
\end{align*}</cmath>
 +
Therefore, the answer is <math>\boxed{\textbf{(D)}}.</math>
 +
 +
More generally, we have <math>\Delta^k(u_n)=0</math> for all <math>n</math> if <math>k\geq4.</math>
 +
 +
~MRENTHUSIASM
 +
 +
== See Also ==
 +
{{AHSME box|year=1976|num-b=24|num-a=26}}
 +
{{MAA Notice}}

Revision as of 11:22, 7 September 2021

Problem

For a sequence $u_1,u_2\dots$, define $\Delta^1(u_n)=u_{n+1}-u_n$ and, for all integer $k>1, \Delta^k(u_n)=\Delta^1(\Delta^{k-1}(u_n))$. If $u_n=n^3+n$, then $\Delta^k(u_n)=0$ for all $n$

$\textbf{(A) }\text{if }k=1\qquad \\ \textbf{(B) }\text{if }k=2,\text{ but not if }k=1\qquad \\ \textbf{(C) }\text{if }k=3,\text{ but not if }k=2\qquad \\ \textbf{(D) }\text{if }k=4,\text{ but not if }k=3\qquad\\ \textbf{(E) }\text{for no value of }k$

Solution

Note that \begin{align*} \end{align*} Therefore, the answer is $\boxed{\textbf{(D)}}.$

More generally, we have $\Delta^k(u_n)=0$ for all $n$ if $k\geq4.$

~MRENTHUSIASM

See Also

1976 AHSME (ProblemsAnswer KeyResources)
Preceded by
Problem 24
Followed by
Problem 26
1 2 3 4 5 6 7 8 9 10 11 12 13 14 15 16 17 18 19 20 21 22 23 24 25 26 27 28 29 30
All AHSME Problems and Solutions

The problems on this page are copyrighted by the Mathematical Association of America's American Mathematics Competitions. AMC logo.png